0 Daumen
828 Aufrufe
Für welche reellen Zahlen x gilt: |(x+4)/(x-2) | < x?

Ich habe 3 Fälle unterschieden.

1.Fall: x < -4

(x+4)/(x-2) < x (weil ja beide ein negatives Vorzeichen hätten. habe ich es direkt als positiven Bruch jetzt geschrieben)

--> 0 > x^2-3x-4

Dann Faktorisieren bzw. mit pq-Formel die Nullstellen ausrechnen

--> x1 = 4 v x2 = -1

--> x^2-3x-4 = (x-4)(x+1)

So habe ich das bei den anderen beiden Fällen auch immer gemacht.

Fall 2: -4 <= x < 2

(x+4)/-(x-2) < x

--> 0 > x^2 -x +4

Wenn ich das dann mit der pq- Formel auflöse, erhalte ich etwas Negatives unter der Wurzel, weshalb ich das ja dann weg lassen kann.

Fall 3: x > 2

Läuft analog zu Fall 1.

Ich weiß jetzt aber ehrlich gesagt nicht, wie ich dann jeweils weiter machen soll, sodass ich am Ende auf eine Lösungsmenge komme.
Avatar von
In der Eingangsfrage soll stehen: |(x+4)/(x-2)| < x!!
EDIT: Betrag ergänzt.

Du kannst übrigens von Anfang an x< 0 und x = 2 ausschliessen, da der Betrag nicht neg. sein kann und unter dem Bruchstrich keine 0 stehen darf.

Fälle:

0 ≤ x < 2 und 2 < x.
Warum kann das denn unterm Bruchstrich nicht negativ werden?? Also, dass es nicht 2 sein darf leuchtet mir ja ein und das hatte ich ja auch berücksichtigt in meinen Fällen, weil es ja bei mir echt > 2 sein muss.

Aber ein negativer bruch klappt doch, oder nicht?

Als Beispiel wähle ich mal -3, dann steht da doch:

|(-3+4)/(-3-2)| = |1/-5|= 1/5, also geht doch x<0 auf jeden Fall.
Ach, Fehler entdeckt!! Dann wäre ja 1/5 < -3 und das geht selbstverständlich nicht.

Aber dann geht das bei 0 ja auch nicht, denn: |(0+4)/(0-2)| = |-2| = 2 < 0 (keine wahre Aussage)

Aber wie löse ich denn nun weiter da oben bei mir auf, sodass ich zu meiner Lösungsmenge komme??
Durch reines Ausprobieren komme ich jetzt darauf, dass x echt > 4 sein muss, damit das klappen kann.
EDIT: Fehler gefunden. ---> Antwort.
Ja, also beim Fall x > 2:

Da müsste dann ja stehen: 0 < x^2 - 3x - 4 und nicht +4 --> somit existieren auch wieder Nullstellen an den Stellen x1=4 v x2=1.

Jetzt kann ich das Ganze ja auch als  0 < (x-4)(x+1)  schreiben (da ja: (x-4)(x+1) = x^2-3x-4 ist).

Kann ich jetzt nochmal Fälle unterscheiden und zwar so??:

1.) x-4 > 0 oder x+1 > 0 --> da xecht > 2 vorgegeben ist geht nur das, da x>4 insb. >2 ist??

2.) x-4 > 0 oder x+1 <0

3.) x-4 < 0 oder x+1 < 0

4.) x-4 <0 oder x+1 > 0

 da xecht > 2 vorgegeben ist geht nur das, da x>4 insb. >2 ist??

Ja. Genau.

Hier noch die beiden Parabeln im Koordinatensystem:

Aha!! Ok, super. Vielen Dank dir! :)

1 Antwort

+1 Daumen

|(x+4)/(x-2)| < x

Fall 0 ≤ x < 2

| (x+4) / (x - 2) | = ( x+4) / ( 2-x ) < x          |* (2-x)

x + 4 < (2-x) x = 2x - x^2

x^2 - x + 4 < 0

x1,2 = 1/2 ( 1 ± √(1 - 16))  

Hat keine Nullstelle -15 unter der Wurzel

Da die Parabel mit der Gleichung y = x^2 - x + 4  nach oben geöffnet ist, verläuft sie ganz über der x-Achse. Die Ungleichung ist nie erfüllt.

Fall x > 2

| (x+4) / (x - 2) | = ( x+4) / ( x-2 ) < x          |* (2-x)

x + 4 < (x-2) x = x^2 - 2x

0 < x^2 - 3x - 4  

x 1,2 = 1/2 ( 3 ± √(9 + 16))  

x1 = 1/2 ( 8) = 4

x2 = 1/2 (-2) = -1

Diese Parabel verläuft ab x = 4 über der x-Achse.

D. h. nun x > 4

L = {x | x >4} 

 

https://www.wolframalpha.com/input/?i=%7C%28x%2B4%29%2F%28x-2%29%7C+%3C+x

Avatar von 162 k 🚀

Ein anderes Problem?

Stell deine Frage

Willkommen bei der Mathelounge! Stell deine Frage einfach und kostenlos

x
Made by a lovely community